bgo

New Member
ارسال ها
276
لایک ها
397
امتیاز
0
#22
پاسخ : از مرحله دو تا جهانی

با فرض
و
اعدادی حقیقی باشند ثابت


(ویرایش شد)
نمی دونم دارم جوب میزنم یا نه ولی فک کنم اینم غلطه چون مثلن لاندا رو یک بذارید بعد a رو هم یک بذارید حالا b,c رو برابر بگیرید و خیلی کوچیک جمله اول که میره به سمت یک جمله دوم و سوم هم با هم برابرن و میشن
که چون a=1 پس میشه b رو اونقدر کوچیک گرفت که این آقاهه از هر چیزی کمتر شه............:3:
 

goodarz

Well-Known Member
ارسال ها
1,026
لایک ها
1,120
امتیاز
113
#23
پاسخ : از مرحله دو تا جهانی

با فرض
و
اعدادی حقیقی باشند ثابت


(ویرایش شد)
یه سوال خیلی خیلی شبیه به این سوال, تعمیم IMO 2001 هست که میگه
برای هر
 
لایک ها bgo

fereidoon

Active Member
ارسال ها
447
لایک ها
132
امتیاز
43
#24
پاسخ : از مرحله دو تا جهانی

سوال اصلی همینیه که گودرز نوشته،احتمالا ali_irysc سوال رو از کتاب صفا برداشته،چون اونجا هم همین اشتباه رو کرده!
 

hkh74

New Member
ارسال ها
213
لایک ها
392
امتیاز
0
#25
پاسخ : از مرحله دو تا جهانی

ببخشید دارم در مورد سوال قبل حرف می زنم، چون سوال حل شده من ایده ای که می گفتم رو میذارم (البته متاسفانه من راه دوم ali_irysc رو متوجه نشدم ولی استقرا درسته) :

رو به صورت
تعریف می کنیم که
. حکم مسئله معادل با اینه که:



از فرض هم نتیجه میگیریم:


که تبدیل میشه به مسئله ی کتاب (104 مسئله ی تئوری اعداد) که با استقرا ثابت میشه.
 

ali_irysc

New Member
ارسال ها
319
لایک ها
294
امتیاز
0
#26
پاسخ : از مرحله دو تا جهانی

مطمئنین درسته؟؟؟؟؟؟؟؟
الان اگه (a,b,c)=(1,1,0) باشه که خب غلط میشه که..........:3:
اقا من درست این سوال رو تو یه مقاله دیدم اگه

ثابت کنید
 

bgo

New Member
ارسال ها
276
لایک ها
397
امتیاز
0
#27
پاسخ : از مرحله دو تا جهانی

اقا من درست این سوال رو تو یه مقاله دیدم اگه

ثابت کنید
خب اینکه با اون فرق داره اونجا نوشته بودید 4 تا سیگما ab............:4:
 

ali_irysc

New Member
ارسال ها
319
لایک ها
294
امتیاز
0
#28
پاسخ : از مرحله دو تا جهانی

سلام اولا که پ نه پ
دادا من این دو تا سوال غلط رو از کتاب جبر صفا برداشتم
 

ali math

New Member
ارسال ها
79
لایک ها
19
امتیاز
0
#29
پاسخ : از مرحله دو تا جهانی

در مثلث abcدایره ی محاطی مثلث را میکشیم تا بر bcدرdمماس باشد.adدایره ی مذکور را در kقطع میکند.I1,i2مراکز دایره ی محاطی مثلث های bkd,ckdمیباشد.ثابت کنیدi1i2باbcموازیه.
این سوال فکر کنم مال short listولی نمیدونم چه سالی.در ضمن خودم حل نکردم.:confused:
 

hkh74

New Member
ارسال ها
213
لایک ها
392
امتیاز
0
#30
پاسخ : از مرحله دو تا جهانی

در مثلث abcدایره ی محاطی مثلث را میکشیم تا بر bcدرdمماس باشد.adدایره ی مذکور را در kقطع میکند.I1,i2مراکز دایره ی محاطی مثلث های bkd,ckdمیباشد.ثابت کنیدi1i2باbcموازیه.
این سوال فکر کنم مال short listولی نمیدونم چه سالی.در ضمن خودم حل نکردم.:confused:
صورت سوال اشتباهه... شکلش رو کشیدم ولی موازی نبود.

 

yasamin.m

New Member
ارسال ها
43
لایک ها
4
امتیاز
0
#31
پاسخ : از مرحله دو تا جهانی

یه مثلث
داریم و یه نقطه به نام p درون مثلث طوری انتخاب میکنیم که داشته باشیم



و d را مرکز داره محاطیه مثلث apb و e را مرکز دایره محاطیه مثلث apc در نظر بگیرید ثابت کنید سه خطه bd و ceو ap همرسن
(سوالم ماله imo 1996 ه)
 

ash1374

New Member
ارسال ها
253
لایک ها
422
امتیاز
0
#32
پاسخ : از مرحله دو تا جهانی

یه مثلث
داریم و یه نقطه به نام p درون مثلث طوری انتخاب میکنیم که داشته باشیم



و d را مرکز داره محاطیه مثلث apb و e را مرکز دایره محاطیه مثلث apc در نظر بگیرید ثابت کنید سه خطه bd و ceو ap همرسن
(سوالم ماله imo 1996 ه)
جای eوdباید عوض شود. لطفا صورت سوال را تصحیح کنید.
 

yasamin.m

New Member
ارسال ها
43
لایک ها
4
امتیاز
0
#33
پاسخ : از مرحله دو تا جهانی

نخیر سوال درسته درسته مشکلی هم وجود نداره :116::116:
 

goodarz

Well-Known Member
ارسال ها
1,026
لایک ها
1,120
امتیاز
113
#34
پاسخ : از مرحله دو تا جهانی

یه مثلث
داریم و یه نقطه به نام p درون مثلث طوری انتخاب میکنیم که داشته باشیم



و d را مرکز داره محاطیه مثلث apb و e را مرکز دایره محاطیه مثلث apc در نظر بگیرید ثابت کنید سه خطه bd و ceو ap همرسن
(سوالم ماله imo 1996 ه)
ثابت کنید p روی دایره آپولونیوس راس A هستش...
 

ali math

New Member
ارسال ها
79
لایک ها
19
امتیاز
0
#36
پاسخ : از مرحله دو تا جهانی

در مثلث abcازhمرکز ارتفاعی خطی رسم کردیم تاac,abرادرp,qقطع کند کهap=aq.ثابت کنیدhmعمود بر وتر مشترک دو دایره یapq,abcاست(mوسطbc)
راستی در مورد سوال قبل هم که گذا شتم ممنون چون اگه کسی اشتباهشو نمیگرفت حالا حالا ه.... .
در ضمن این سوال کاملا درسته(دیگه نترسید):53:
 

goodarz

Well-Known Member
ارسال ها
1,026
لایک ها
1,120
امتیاز
113
#37
پاسخ : از مرحله دو تا جهانی

AoPS Forum - Hard to approach it ! • Art of Problem Solving

فرض کنید T محل برخورد دوم اون دوتا دایره محیطی باشه. ثابت کنید TH نیمساز PTQ هست و نتیجه بگیرین ATH=90. حالا ثابت کنید MH از T رد میشه!!!!!
 

mahanmath

New Member
ارسال ها
898
لایک ها
701
امتیاز
0
#38
پاسخ : از مرحله دو تا جهانی

در مثلث abcازhمرکز ارتفاعی خطی رسم کردیم تاac,abرادرp,qقطع کند کهap=aq.ثابت کنیدhmعمود بر وتر مشترک دو دایره یapq,abcاست(mوسطbc)
BE ، CF رو ارتفاع‌های مثلث بگیرید . دایره AEHF رو هم
بگیرید
رو هم دایره محیطی‌ بگیرید .



پس نسبت قوت P,Q به این ۲ دایره یکیه ، پس طبقه یک لم معروف درباره قوت (در هندسه مسطحه Kurt هم برسی‌ شده) P,Q با محل تقاطع اون ۲ دایره‌ها هم دایرن . پس کافیه نشون بدیم که HM بر وتر مشترک این ۲ دایره عمود هست که اینم بدیهیه چون H ، M و تقاطع غیر از A
,
با هم هم خط هستند !

(برای یه اثبات سریع برای این هم خطی‌ ، میتونید بگید که M روی خط BC و روی دایره نه‌-نقطه هست پس بعد از انعکاس به مرکز H روی تقاطع اون‌ها هست . اما تصویر BC همون دایره
و تصویر نه‌-نقطه دایره محیطی‌ یعنی‌
می‌شه .)

 

mahanmath

New Member
ارسال ها
898
لایک ها
701
امتیاز
0
#39
پاسخ : از مرحله دو تا جهانی

یه راه مسخره تر پیدا کردم ، در واقع کلشو با انعکاس گفتم ، اگه خواستین اون رو هم بنویسم .:1:
 
بالا